LSAT and Law School Admissions Forum

Get expert LSAT preparation and law school admissions advice from PowerScore Test Preparation.

 Administrator
PowerScore Staff
  • PowerScore Staff
  • Posts: 8916
  • Joined: Feb 02, 2011
|
#40157
Complete Question Explanation
(The complete setup for this game can be found here: lsat/viewtopic.php?t=8560)

The correct answer choice is (E)

This is the infamous Rule Substitution question, a staple in the Logic Games section as of late. Considering their growing popularity, tackling such questions is no longer merely optional: it is imperative to have a workable strategy in place.

The question stem asks you to replace the second rule of the game, according to which neither X nor Y can be earlier than Z:
PT73_Game_#2_#13_diagram 1.png
Our job is to substitute that rule with a logically equivalent condition, which would have the exact same effect on the order in which the speeches are given. While the wording of the correct answer can be difficult-to-impossible to predict, the four incorrect answer choices will either present rules that were not part of the original rule set (also known as Additional Effects conditions), or else they will only partially constrain the variables in the rule being substituted (i.e. Partial Match conditions).

Since Additional Effects are easier to spot, look for answer choices that contain such conditions first. You should immediately notice, for instance, that answer choice (A) adds a condition extraneous to the original rule set, because it stipulates that L must be at 3 PM. This is not required by the original rule set, given that L can also be at 2 PM (see the local diagram for Question #10).

We are left with answer choices (B), (C), (D), and (E).

Answer choice (B): This rule prohibits M from being earlier than Z. In other words, M must be given either at the same time as Z, or else later than Z:
  • ..... ..... ..... ..... ..... Z======M
This condition is true given our original diagram, because Z must be at 1 PM (main inference), and thus all other speeches are either given later than Z, or at the same time as Z. However, this restraint does not prevent either Y or X from being earlier than Z, which the original rule did. For instance, the following setup would be possible if we substituted the second rule for the rule in answer choice (B):
PT73_Game_#2_#13_diagram 2.png
Clearly, answer choice (B) is a Partial Match condition, and is therefore incorrect.

Answer choice (C) is also incorrect. Here, the stipulation is that either X or Y be after Z. This is clearly an implication of the rule we need to substitute, because it is impossible for all three speeches to be given at the same time, and so at least one of X or Y would have to be given after Z. However, our job is to ensure that neither X nor Y can be earlier than Z, which this rule cannot do. For instance, the following solution would be possible if we are allowed to place just one of X or Y after Z:
PT73_Game_#2_#13_diagram 3.png
Therefore, answer choice (C) also contains a Partial Match, and must be eliminated.

Answer choice (D) states that either X or Y (or both) must be at 2 PM; in other words, at least one of them must be at 2 PM. This is clearly true given the original rule set, because X and Y cannot both be given at 1 PM without violating the second rule, and there is only one speech being given at 3 PM. However, requiring just one of X or Y to be at 2 PM still allows the other one to be earlier than Z, in violation of the rule being substituted. For instance, the following hypothetical setup would be possible with this restraint in effect:
PT73_Game_#2_#13_diagram 4.png
Thus, answer choice (D) also contains a Partial Match, and must be eliminated.

Answer choice (E) is the correct answer choice. The condition requiring Z to be at 1 PM is consistent with the original diagram, as it is the main inference in the game. Answer choice (E), therefore, does not contain an Additional Effects rule. The better question is: does answer choice (E) contain a Partial Match? It does not, because if Z is at 1 PM, then neither X nor Y can be earlier than Z. Thus, the effect of the new condition is identical to that of the original rule, and answer choice (E) is correct.
You do not have the required permissions to view the files attached to this post.
 Rosaline
  • Posts: 17
  • Joined: Apr 29, 2018
|
#48252
Hi. I generally struggle with Rule Sub questions. Is there a posting could someone please offer me some guidance on the best way to approach these types of questions? Thanks!
User avatar
 Jonathan Evans
PowerScore Staff
  • PowerScore Staff
  • Posts: 726
  • Joined: Jun 09, 2016
|
#48818
Hi, Rosaline,

Yes, these topics are covered in depth in the Logic Games Bible (Page 529-538 of the 2018 edition) and are discussed more briefly in a supplemental module to Chapter 11 of the full-length course.

Many (if not most) LSAT students find these questions challenging. They can be both time-consuming and difficult to evaluate, a deadly combo, especially when you think you're almost done with a game.

To summarize, the basic process is as follows:
  1. Isolate the suspended rule. What are its effects?
  2. How does this rule connect to other rules or inferences? How does it interact with the rest of the game? Which variables are involved?
  3. Keeping these effects and connections in mind, move to the answer choices. Which answer choices forces an identical outcome. You have to be sure that the answer both forces the same effects without adding additional effects into the mix. The correct answer will have identical effects to the replaced rule.
Don't be discouraged! These are tough, but if you are methodical about the approach above, you can confidently find the correct answer. Please follow up with further questions about any specific rule substitution questions you find particularly tricky.

I hope this helps!
 Rosaline
  • Posts: 17
  • Joined: Apr 29, 2018
|
#48906
I took the class, is there somewhere in the supplemental homework that tackles rule substitutions specifically?
User avatar
 Jonathan Evans
PowerScore Staff
  • PowerScore Staff
  • Posts: 726
  • Joined: Jun 09, 2016
|
#48971
Hi, Rosaline,

In our course material, this topic is covered in depth in the separate Advanced LSAT Logic Games course. More information is available here:

https://www.powerscore.com/lsat/courses/advanced/

This specific topic goes slightly beyond the purview of our standard course curriculum. If you wish to read a detailed explanation without registering for an additional course, the Logic Games Bible unit on the subject (Chapter 9, noted above), is probably your best bet.

I am also happy to answer any questions about any specific rule substitution problem, including this one!

Please follow up with further questions.

Thank you!

Get the most out of your LSAT Prep Plus subscription.

Analyze and track your performance with our Testing and Analytics Package.